0 Daumen
3k Aufrufe

Hey:)


wie Beweise ich diese beiden Teilaufgben?:)

Bild Mathematik

Avatar von

$$(a)\quad\int_0^af(x)\,dx\ge af(a)\ge0$$


$$(b)\quad|f(\xi)|>\alpha\Rightarrow|f(x)|>\alpha\quad\text{fuer alle $x$ aus einer ganzen Umgebung von $\xi$}$$

Wie kommst du genau auf die a?

Weil wenn man doch a gegen unendlich laufen lässt, ist doch der mittlere Teil größer

$$  \int_0^a f(x) \ dx = a \ f(\xi)  $$ mit \( 0 \le \xi \le a \) Da \( f \) monoton fallend ist, folgt \( f(\xi) \ge f(a) \)

 f ist monoton fallend, sei lim f(x)= a, a>0

lim (z-> unendlich) Integral von 0 bis z a*dx = lim a*z, da a>0 divergiert das Integral


Weil aber ein Grenzwerz existiert, weil f monoton und beschränkt (f(x)>=0) existiert  ein Grenzwert in R.


Also muss a=0 sein.


Passt das so?

Ich hätte so argumentiert. Wenn \( \lim_{x \to \infty} f(x) = M > 0 \) gilt, folgt
$$ F(a) =  \int_0^a f(x) \ dx \ge a \ f(a) \ge a \ M $$ wegen der Monotonie und und für \( a \to \infty \) sieht man das \( F(a) \) größer als eine beliebige Schranke wird und somit das Integral nicht konvergent ist. Deshalb mus \( \lim_{x \to \infty } f(x) = 0 \) gelten.

Nochwas, schreib doch Deine Fragen so auf, dass man die Formeln auch erkennen kann, dass es geht siehst Du ja hier und ein bisschen TeX ist ja nicht schwer.

1 Antwort

0 Daumen
 
Beste Antwort

Ich hätte so argumentiert. Wenn \( \lim_{x \to \infty} f(x) = M > 0 \) gilt, folgt
$$ F(a) =  \int_0^a f(x) \ dx \ge a \ f(a) \ge a \ M $$ wegen der Monotonie und und für \( a \to \infty \) sieht man das \( F(a) \) größer als eine beliebige Schranke wird und somit das Integral nicht konvergent ist. Deshalb mus \( \lim_{x \to \infty } f(x) = 0 \) gelten.

Nochwas, schreib doch Deine Fragen so auf, dass man die Formeln auch erkennen kann, dass es geht siehst Du ja hier und ein bisschen TeX ist ja nicht schwer.

Avatar von 39 k

Ein anderes Problem?

Stell deine Frage

Willkommen bei der Mathelounge! Stell deine Frage einfach und kostenlos

x
Made by a lovely community